0
$\begingroup$

Let $A\subseteq\mathbb{R}$ be non-empty and bounded. Also, $\forall x\in A$, $\exists\epsilon>0$ such that $(x-\epsilon,x+\epsilon)\subseteq A$. Prove that $A$ is a finite or countable union of pairwise disjoint open intervals.

I'm aware that this question has many different proofs, but I'm interested in knowing how one would go about proving it in this particular way.

For $q\in\mathbb{Q}\cap A$, define

$\alpha(q):= \text{inf}\{x\in\mathbb{R}:(x,q]\subseteq A\}$; $\beta(q):= \text{sup}\{x\in\mathbb{R}:[q,x)\subseteq A\}$

First, why do $\alpha$ and $\beta$ exist and $\alpha<\beta$? Then, how can one show that $I_q:=(\alpha(q),\beta(q))\subseteq A$? Lastly, how can one prove that

  • $\bigcup_{q\in\mathbb{Q}\cap A}I_q=A$
  • $\forall q, s\in\mathbb{Q}\cap A$, either $I_q=I_s$ or $I_q\cap I_s=\emptyset$

Thanks in advance.

$\endgroup$
3
  • $\begingroup$ Essentially, you seek to prove that any open subset of $\mathbb R$ is the union of at most countably many disjoint intervals. You should phrase it as such $\endgroup$ Oct 8, 2018 at 18:53
  • $\begingroup$ $\alpha$ and $\beta$ exist and are finite because $A$ is bounded. $A$ contains some $\epsilon$-neighborhood around $q$, so $\alpha \leq q - \epsilon < q + \epsilon \leq \beta$. You haven't defined $I_q$. $\endgroup$
    – user169852
    Oct 8, 2018 at 18:56
  • $\begingroup$ I see. I have defined $I_q$ now. Can I extend that reasoning to show it's contained in $A$? $\endgroup$
    – user539807
    Oct 8, 2018 at 19:46

1 Answer 1

1
$\begingroup$

Okay, so:

Firsly, let's take any $q\in Q\cap A$. We know that $ q \in A$, so there exists $\epsilon_q$, such: $(q-\epsilon_q;q+\epsilon_q) \subseteq A$.

Now, we're ready to look at $\alpha(q),\beta(q)$. Why do they exist? A is bounded, $[q-\frac{\epsilon_q}{2}; q] \subseteq A $, and $[q;q + \frac{\epsilon_q}{2}] \subseteq A$. It is clear now, that both $\alpha(q),\beta(q)$ exists and $\alpha(q) < \beta(q)$.

I suppose u denote $I_q = (\alpha(q),\beta(q)) $.

$I_q \subseteq A$ because $\alpha(q) \ (\beta(q))$ by definition are such that $(\alpha(q),q] \subseteq A$ and $[q,\beta(q)) \subseteq A \ \ \ \ (*)$

To prove 1 and 2:

1) I'll show two inclusions. Let's pick any $x \in \bigcup_{q\in Q \cap A} I_q $. I'll show that $x \in A$. Clearly for particular $q_x$, $x \in I_{q_x}$, that means $ x \in (\alpha(q_x),\beta(q_x)) $ and by $(*)$ we get $x \in A$

Now let's pick any $y \in A$. There exists $ \epsilon_y $ such that $(y-\epsilon_y;y+\epsilon_y) \subseteq A $. In any interval, there is $ p \in Q $, $p \in (y-\epsilon_y;y+\epsilon_y), p>y $. Let's look at $I_p = (\alpha(p),\beta(p)) $. I'll show that $\alpha(p)<y$, which is obvious because by definition of $\alpha(p)$, there must holds inequality: $\alpha(p) \leq y-\epsilon_y < y $.

So $ y \in (\alpha(p),p) \subseteq I_p \in \bigcup_{q \in Q \cap A} I_q $

2) If $I_q = I_s$ everything's good, if no, let's assume that $I_q \cap I_s \neq \emptyset$

Just to fix something, say $q<s$ ($s<q$ case is really the same).

Clearly then $\beta(q) < \beta(s) $, but $\beta(q) \in (\alpha(s),\beta(s))$ that means $\beta(q)$ cannot be $ \sup\{x\in R: [q,x) \in A\} $ (because $[q,\beta(q)) \in A , (\alpha(s), \beta(s)) \in A , \alpha(s) < \beta(q) < \beta(s)$ (otherwise it really would be $I_q \cap I_s = \emptyset $)

So,there we have a problem, cause whole $[q,\beta(s)) \subseteq A$, and that means $\beta(q)$ wasn't supremum. Contrary

$\endgroup$
2
  • $\begingroup$ Thank you. This was extremely clear. What can we say about "finite or countable"? i.e., why does this suffice as a proof for the theorem? $\endgroup$
    – user539807
    Oct 8, 2018 at 23:35
  • $\begingroup$ We're looking at rationals that are in the set A, so the union cannot be uncountable ( rationals are countable and we're looking only at some of them.) $\endgroup$
    – Presage
    Oct 9, 2018 at 5:16

You must log in to answer this question.

Not the answer you're looking for? Browse other questions tagged .